Kitty Kat

Pataasin ang iyong marka sa homework at exams ngayon gamit ang Quizwiz!

After surgery for repair of a myelomeningocele, the nurse places the infant in a side-lying position with the head slightly elevated. What is the main reason the nurse places the infant in this position after this particular surgery? 1. To prevent aspiration 2. To promote respiration 3. To reduce intracranial pressure 4. To maintain cleanliness of the suture site

3. To reduce intracranial pressure

A client's cardiac monitor shows a PQRST wave for each beat and indicates a rate of 120 beats per minute. The rhythm is regular. The nurse concludes that the client is experiencing what? 1. Atrial fibrillation 2. Sinus tachycardia 3. Ventricular fibrillation 4. First-degree atrioventricular block

2. Sinus tachycardia

A client's relative asks the nurse what a cataract is. Which explanation should the nurse provide? 1. An opacity of the lens 2. A thin film over the cornea 3. A crystallization of the pupil 4. An increase in the density of the conjunctiva

1. An opacity of the lens

What characteristic is most essential for the nurse caring for a client undergoing mental health care? 1. Empathy 2. Sympathy 3. Organization 4. Authoritarianism

1. Empathy

Which information may be obtained by palpation? Select all that apply. 1. Turgor 2. Bruises 3. Texture 4. Lesions 5. Moisture content 6. Tissue integrity

1. Turgor 3. Texture 4. Lesions 5. Moisture content

A client reports swelling of the scrotum with no pain. Which condition does the nurse anticipate in the client? 1. Chancroid 2 .Hydrocele 3. Spermatocele 4. Incarcerated hernia

2 .Hydrocele

A couple seeking genetic counseling are heterozygous carriers of Tay-Sachs disease. They ask the nurse what the chances are that each of their children will inherit the disease. The nurse responds that the probability is what? 1. 0% 2. 25% 3. 50% 4. 100%

2. 25%

The primary healthcare provider diagnoses placenta previa in a primiparous client. What does this indicate to the nurse regarding the condition of the placenta? 1. Infarcted 2. Low-lying 3. Immaturely developed 4. Separating prematurely

2. Low-lying

A nurse anticipates that most clients with phobias will use which defense mechanisms? 1. Dissociation and denial 2. Introjection and sublimation 3 Projection and displacement 4 Substitution and reaction formation

3 Projection and displacement

A client feeling increasingly tired seeks medical care. Type 1 diabetes is diagnosed. What causes increased fatigue with type 1 diabetes? 1. Increased metabolism at the cellular level 2. Increased glucose absorption from the intestine 3, Decreased production of insulin by the pancreas 4. Decreased glucose secretion into the renal tubules

3, Decreased production of insulin by the pancreas

A client with adrenal insufficiency reports feeling weak and dizzy, especially in the morning. What should the nurse determine is the most probable cause of these symptoms? 1. A lack of potassium 2. Postural hypertension 3. A hypoglycemic reaction 4. Increased extracellular fluid volume

3. A hypoglycemic reaction

When assessing a client, the nurse auscultates a murmur at the second left intercostal space (ICS) along the sternal border. This reflects sound from which valve? 1. Aortic 2. Mitral 3. Pulmonic 4. Tricuspid

3. Pulmonic

While completing an assessment, the nurse finds that a client has decreased thickness and excessive dryness of the epidermis. Which clinical finding is associated with this skin assessment? 1. Skin tears 2. Skin cancer 3. Skin fragility 4. Skin hyperplasia

3. Skin fragility

Which type of joint is present in between the client's tarsal bones? 1. Pivot joint 2. Hinge joint 3. Saddle joint 4. Gliding joint

4. Gliding joint

A client visits a primary healthcare provider with a report of burning and a sharp pain in the sole of the foot that intensifies in the morning. Which abnormal condition does the nurse anticipate in the client? 1. Torticollis 2. Pes planus 3. Tenosynovitis 4. Plantar fasciitis

4. Plantar fasciitis

During the first session of a therapy group, one of the clients asks, "What's supposed to happen in this group?" What is the most appropriate response by the nurse facilitator? 1. "First I'd like for you to tell me what you want to happen." 2. "This is your group, and your participation will largely determine what happens." 3. "The purpose of this group is to examine the way each of you interacts with the others." 4. "You and the others are supposed to discuss any reality-based concerns you have about your illness."

1. "First I'd like for you to tell me what you want to happen."

What is most appropriate for a nurse to say when interviewing a newly admitted depressed client whose thoughts are focused on feelings of worthlessness and failure? 1. "Tell me how you feel about yourself." 2. "Tell me what has been bothering you." 3. "Why do you feel so bad about yourself?" 4. "What can we do to help you while you're here?"

1. "Tell me how you feel about yourself."

A client is admitted to the birthing unit with uterine tenderness and minimal dark-red vaginal bleeding. She has a marginal abruptio placentae. The priority assessment includes fetal status, vital signs, skin color, and urine output. What additional assessment is essential? 1. Fundal height 2. Obstetric history 3. Time of the last meal 4. Family history of bleeding disorders

1. Fundal height

Which is a clinical manifestation of the Landouzy-Déjérine type of muscular dystrophy (MD)? 1. Loss of hearing 2. Cardiomyopathy 3. Respiratory failure 4. Mental impairment

1. Loss of hearing

Which clinical manifestation occurs in a client with adrenal insufficiency? 1. Vitiligo 2. Moon face 3. Hypertension 4. Truncal obesity

1. Vitiligo

An older adult in an acute care setting is having urinary incontinence. Which interventions would help the client? Select all that apply. 1. Provide nutritional support 2 Provide voiding opportunities 3. Avoid indwelling catheterization 4. Provide beverages and snacks frequently 5. Promote measures to prevent skin breakdown

2 Provide voiding opportunities 3. Avoid indwelling catheterization 5. Promote measures to prevent skin breakdown

A client is admitted for treatment of partial- and full-thickness burns of the entire right lower extremity and the anterior portion of the right upper extremity. The nurse performs an immediate appraisal, using the rule of nines. Which percentage of body surface area burned will the nurse record? 1. 18% 2. 22.5% 3. 27% 4. 36.5%

2. 22.5%

A hospitalized psychiatric client with the diagnosis of histrionic personality disorder demands a sleeping pill before going to bed. After being refused the sleeping pill, the client throws a book at the nurse. What does the nurse recognize this behavior to be? 1. Exploitive 2. Acting out 3. Manipulative 4. Reaction formation

2. Acting out

A client with varicose veins is scheduled for surgery. Which clinical finding does the nurse expect to identify when assessing the lower extremities of this client? 1. Pallor 2. Ankle edema 3. Yellowed toenails 4. Diminished pedal pulses

2. Ankle edema

While assessing the skin of a light-skinned client, the nurse concludes that the client has ecchymosis. Which skin color variation would confirm this diagnosis? 1. Gray color 2. Dark red color 3. Deep brown color 4. White color

2. Dark red color

According to Kübler-Ross, during which stage of grieving are individuals with serious health problems most likely to seek other medical opinions? 1. Anger 2. Denial 3. Bargaining 4. Depression

2. Denial

A nurse is providing discharge teaching for a client who recently had surgery for an abdominal perineal resection of the colon and the creation of a colostomy. Which condition will the nurse share with the client for when to call the healthcare provider immediately? 1. Intestinal cramps during fluid inflow 2. Difficulty inserting the irrigation tube 3. Passage of flatus during expulsion of feces 4. An inability to complete the procedure in one hour

2. Difficulty inserting the irrigation tube

What nursing action will most help a client obtain maximum benefits after postural drainage? 1. Administer oxygen as needed. 2. Encourage coughing deeply. 3. Place the client in a sitting position. 4. Encourage the client to rest for a half hour

2. Encourage coughing deeply

The nurse in the postanesthesia care unit is caring for a client who had a left-sided pneumonectomy. Which goal is priority? 1. Replace blood loss 2. Maintain ventilatory exchange 3. Maintain closed chest drainage 4. Replace supplemental oxygenation

2. Maintain ventilatory exchange

Which should the nurse encourage for a school-age client diagnosed with a chronic illness to enhance a sense of accomplishment? 1. Wearing make-up 2. Making up missed work 3. Participating in sports activities 4. Participating in creative activities

2. Making up missed work

During a physical assessment, the nurse notes cutaneous fibromas and Lisch nodules (yellow elevations) on a client's irises. What genetic condition might this client have? 1. Phenylketonuria 2. Neurofibromatosis 3. Huntington disease 4. Myotonic dystrophy

2. Neurofibromatosis

During a newborn assessment the nurse identifies the absence of the red reflex in the eyes. What should the nurse's next action be? 1. Rinse the eyes with sterile saline 2. Notify the primary healthcare provider 3. Expect edema to subside within a few days 4. Conclude that this is a result of the prescribed eye prophylaxis

2. Notify the primary healthcare provider

The healthcare provider prescribes a progressive exercise program that includes walking for a client with a history of diminished arterial perfusion to the lower extremities. The nurse explains to the client what to do if leg cramps occur while walking. Which instruction did the nurse give the client? 1. Chew one aspirin twice a day. 2. Stop to rest until the pain resolves. 3. Walk more slowly while pain is present. 4. Take one nitroglycerin tablet sublingually.

2. Stop to rest until the pain resolves.

A client is admitted to the hospital with the diagnosis of severe anxiety. What should the nurse's plan of care for a client with an anxiety disorder include? 1. Promoting the suppression of anger by the client 2. Supporting the verbalization of feelings by the client 3. Encouraging the client to limit anxiety-related behaviors 4. Restricting the involvement of the client's family during the acute phase

2. Supporting the verbalization of feelings by the client

A young client who has just lost her first job comes to the mental health clinic very upset and says, "I just start crying without any reason and without any warning." How should the nurse respond initially? 1. "Do you know what makes you cry?" 2. "Most of us need to cry from time to time." 3. "Crying unexpectedly can be very upsetting." 4. "Are you having any other problems at this time?"

3. "Crying unexpectedly can be very upsetting."

A client develops a nonhealing ulcer of a lower extremity and complains of leg cramps after walking short distances. The client asks the nurse what causes these leg pains. Which would be the best response by the nurse? 1. "Muscle weakness occurs in the legs because of a lack of exercise." 2. "Edema and cyanosis occur in the legs because they are dependent." 3. "Pain occurs in the legs while walking because there is a lack of oxygen to the muscles." 4. "Pressure occurs in the legs because of vasodilation and pooling of blood in the extremities."

3. "Pain occurs in the legs while walking because there is a lack of oxygen to the muscles."

A pregnant woman who is in the third trimester arrives in the emergency department with vaginal bleeding. She states that she snorted cocaine approximately 2 hours ago. Which complication does the nurse suspect as the cause of the bleeding? 1. Placenta previa 2. Tubal pregnancy 3. Abruptio placentae 4. Spontaneous abortion

3. Abruptio placentae

Windows in the recreation room of the adolescent psychiatric unit have been broken on numerous occasions. After a group discussion one of the adolescents confides that another adolescent client broke them. What should the nurse do when using an assertive intervention instead of aggressive confrontation? 1. Confront the adolescent openly in the group, using a controlled voice and maintaining direct eye contact. 2. Knock on the door of the adolescent's room and ask whether the adolescent would come out to talk about the situation. 3. Approach the adolescent when the client is alone and, after making direct eye contact, inquire about the involvement in these incidents. 4. Use a trusting approach toward the adolescent and imply that the staff doubts the adolescent's involvement but requests a denial for the record

3. Approach the adolescent when the client is alone and, after making direct eye contact, inquire about the involvement in these incidents.

A nurse is caring for a client after abdominal surgery and encourages the client to turn from side to side and to engage in deep-breathing exercises. What complication is the nurse trying to prevent? 1. Metabolic acidosis 2. Metabolic alkalosis 3. Respiratory acidosis 4. Respiratory alkalosis

3. Respiratory acidosis

A client is noted to have thickened toenails that overhang the toes. The registered nurse suspects a fungal infection and instructs the student nurse to examine the fungal infection to confirm the diagnosis. Which action of the student nurse needs correction? 1. Cutting the client's fingernails straight across 2. Using the client's fingernails for assessing capillary refill 3. Using the nail appearance alone for assessing fungal infection 4. Assessing skin next to the nail to determine whether the thick nail is irritating the skin

3. Using the nail appearance alone for assessing fungal infection

A nurse is evaluating scenarios that are based on the responses of several clients. Which statement of a client confirms that he or she has reached the Intimacy versus Isolation stage according to Erikson's theory of psychosocial development? 1. "I donate a large sum of money to the local school every year." 2. "I want to enjoy my motherhood and that's why I am leaving the job." 3. "In the winter of my life, I feel that I do not have anyone to take care of me." 4. "I did so much for my partner but I was dumped for someone more attractive."

4. "I did so much for my partner but I was dumped for someone more attractive."

The nurse provides discharge teaching for a client with a history of hypertension who had a femoropopliteal bypass graft. Which client statement indicates teaching is effective? 1. "I should massage my calves and feet every day." 2. "I should keep my foot elevated when I am in bed." 3. "I should sit in a hot bath for half an hour twice a day." 4. "I should observe the color and pulses of my legs every day."

4. "I should observe the color and pulses of my legs every day."

A primary healthcare provider diagnoses late-stage (tertiary) syphilis in a client. Which statement made by the client supports this diagnosis? 1. "I noticed a wart on my penis." 2. "I have sores all over my mouth." 3. "I've been having a sore throat lately." 4. "I'm having trouble keeping my balance."

4. "I'm having trouble keeping my balance."

Which over-the-counter (OTC) drug is used as the first line of therapy for acne vulgaris in adolescents? 1. Tretinoin 2. Isotretinoin 3. Azithromycin 4. Benzoyl peroxide

4. Benzoyl peroxide

A client is admitted with paresis of the ciliary muscles of the left eye. What function should the nurse expect to be affected? 1. Closing the eyelids 2. Convergence of both eyes 3. Ability to discriminate colors 4. Focusing the lens on near objects

4. Focusing the lens on near objects

A cast is applied to the involved extremity of an infant with talipes equinovarus (clubfoot). How often does the nurse tell the parents to bring their baby back to the clinic for a cast change? 1. Each week 2. Once a month 3. When the cast edges fray 4. If the cast becomes soiled

1. Each week

A client is admitted to the psychiatric unit with the diagnosis of obsessive-compulsive disorder. The client washes her hands more than 20 times a day, and they are raw and bloody. What defense mechanism does the nurse conclude that the client is using to ease anxiety? 1. Undoing 2. Projection 3. Introjection 4 Displacement

1. Undoing

A client describes his delusions in minute detail to the nurse. How should the nurse respond? 1. Changing the topic to reality-based events 2. Continuing to discuss the delusion with the client 3. Getting the client involved in a social project with peers 4. Disputing the perceptions with the use of logical thinking

1. Changing the topic to reality-based events

A nurse is caring for a client with hyperthyroidism. Which laboratory test will be most beneficial in monitoring the effectiveness of drug therapy? 1. Free thyroxine (FT4) 2. Thyroxine (T4), total 3. Free triiodothyronine (FT3) 4. Triiodothyronine (T3), total

2. Thyroxine (T4), total

The nurse is assessing a client with severe nodule-forming rheumatoid arthritis for possible Felty syndrome. Which assessment findings are consistent with Felty syndrome? Select all that apply. 1. Itchy eyes 2. Dry mouth 3. Leukopenia 4. Splenomegaly 5. Photosensitivity

3. Leukopenia 4. Splenomegaly

While auscultating the heart, a healthcare provider notices S3 heart sounds in four clients. Which client is at more risk for heart failure? 1. Child client 2. Pregnant client 3. Older adult client 4. Young adult client

3. Older adult client

A client with liver dysfunction reports bleeding gums. In addition, the nurse observes small facial hemorrhagic lesions. What should the nurse conclude that the client needs? 1. Vitamin C 2. Folic acid 3. Vitamin A 4. Vitamin K

4. Vitamin K

A nurse is counseling the spouse of a client who has a history of alcohol abuse. What does the nurse explain is the main reason for drinking alcohol in people with a long history of alcohol abuse? 1. They are dependent on it. 2. They lack the motivation to stop. 3. They use it for coping. 4. They enjoy the associated socialization

They are dependent on it

A nurse is assessing an 18-month-old toddler with suspected developmental dysplasia of the left hip. In what position should the nurse place the toddler to elicit the Trendelenburg sign? 1. Standing on the affected leg 2. Supine with the back arched 3. Side-lying on the unaffected side 4. Sitting upright with the legs separated

1. Standing on the affected leg

A healthcare provider prescribes oral loperamide (Maalox) and intravenous ranitidine for a client with burns and crushing injuries caused by an accident. The client asks how these medications work. What is the nurse's best response? 1. "They decrease irritability of the bowel." 2. "They limit acidity in the gastrointestinal tract." 3. "They are very effective in clients with multiple trauma." 4. "They work in the way that antiemetics do."

2. "They limit acidity in the gastrointestinal tract."

A client is admitted for a biopsy of a tumor in her left breast. The client states, "I know it can't be cancer, because it doesn't hurt." What is the nurse's most therapeutic response? 1. "Let's hope that it isn't malignant." 2. "What do you know about breast cancer?" 3. "Most lumps in the breast are not malignant." 4. "Has your primary healthcare provider told you that it wasn't cancer?"

2. "What do you know about breast cancer?"

A father asks a nurse for strategies to convince his 5-year-old to wear a helmet while bicycling. What should the nurse suggest to the father? 1. "You should forbid your child from riding a bicycle." 2. "You should wear your helmet while riding your bicycle." 3. "You should limit your child's bicycling to a defined area." 4. "You should tell your child about the risks associated with not wearing a helmet."

2. "You should wear your helmet while riding your bicycle."

A nurse is assessing a toddler with the diagnosis of lead poisoning. What is the most harmful adverse effect that the nurse anticipates? 1. Inadequate nutrition 2. Delayed development 3. Anemia and constipation 4. Renal and skeletal damage

2. Delayed development

Which identity may fail to develop if the adolescent fails to feel a sense of belonging and acceptance? 1. Sexual identity 2. Group identity 3. Family identity 4. Health identity

2. Group identity

A client with chronic kidney disease is receiving medication to manage anemia. Which primary goal should the nurse include in the care plan from this information? 1. Prevention of uremic frost 2. Prevention of chronic fatigue 3. Prevention of tubular necrosis 4. Prevention of dependent edema

2. Prevention of chronic fatigue

Which screening report will help the nurse determine skeletal growth in a child? 1. Electroencephalogram reports 2. Radiographs of the hand and wrist 3. Magnetic resonance imaging (MRI) 41 Denver Developmental Screening Test

2. Radiographs of the hand and wrist

The nurse is assessing a client who has syndrome of inappropriate antidiuretic hormone (SIADH). Which finding in the client is consistent with the diagnosis? 1.Preservation of salt 2. Retention of water 3. Decrease of vasopressin 4. Presence of pedal edema

2. Retention of water

The laboratory reports of a client who underwent a hypophysectomy show an intracranial pressure (ICP) of 20 mmHg. Which action made by the client is responsible for this condition? 1. Drinking lots of water 2. Eating high-fiber foods 3. Bending over at the waist 4. Bending knees when lowering body

3. Bending over at the waist

Which nursing assessment is most important for a large-for-gestational-age (LGA) infant of a diabetic mother (IDM)? 1. Temperature less than 98° F (36.6° C) 2. Heart rate of 110 beats/min 3. Blood glucose level less than 40 mg/dL (2.2 mmol/L) 4. Increasing bilirubin during the first 24 hours

3. Blood glucose level less than 40 mg/dL (2.2 mmol/L)

A female adolescent complains of breast pain. Which antigonadotropic herb may alleviate breast pain by decreasing prolactin levels? 1. Catnip 2. Black haw 3. Bugleweed 4. Chaste tree fruit

3. Bugleweed

Before an amniocentesis, both parents express anxiety about the fetus's safety during the test. Which nursing intervention is most appropriate in promoting the parents' ability to cope? 1. Initiating a parent-primary healthcare provider conference 2. Reassuring them that the procedure is safe 3. Explaining the procedure, step by step 4. Arranging for the father to be present during the test

3. Explaining the procedure, step by step

A nurse is assessing a client for possible malabsorption syndrome. Which stool assessment finding will support this diagnosis? 1. Melena 2. Frank blood 3. Fat globules 4. Currant jelly consistency

3. Fat globules

A nurse working on a mental health unit is caring for several clients who are at risk for suicide. Which client is at the greatest risk for successful suicide? 1. Young adult who is acutely psychotic 2. Adolescent who was recently sexually abused 3. Older single man just found to have pancreatic cancer 4. Middle-age woman experiencing dysfunctional grieving

3. Older single man just found to have pancreatic cancer

The left foot of a client with a history of intermittent claudication becomes increasingly cyanotic and numb. Gangrene of the left foot is diagnosed, and because of the high level of arterial insufficiency, an above-the-knee amputation (AKA) is scheduled. Which response by the client best demonstrates emotional readiness for the surgery? 1. Explains the goals of the procedure 2. Displays few signs of anticipatory grief 3. Participates in learning perioperative care 4. Verbalizes acceptance of permanent dependency needs

3. Participates in learning perioperative care

A client diagnosed with multiple myeloma has been given a poor prognosis. After discharge, the client plans to travel on an airplane and attend sporting events with friends and family. The nurse prepares a discharge teaching plan for this client. What should the plan include? 1. Eliminating travel plans to combat anemia-related fatigue 2. Reinforcing a positive mental attitude to improve prognosis 3. Preventing infection; the client is at risk for leukopenia 4. Restricting fluid intake; the client is at risk for congestive heart failure

3. Preventing infection; the client is at risk for leukopenia

A clinic nurse observes a 2-year-old client sitting alone, rocking and staring at a small, shiny top that she is spinning. Later the father relates his concerns, stating, "She pushes me away. She doesn't speak, and she only shows feelings when I take her top away. Is it something I've done?" What is the most therapeutic initial response by the nurse? 1. Asking the father about his relationship with his wife 2. Asking the father how he held the child when she was an infant 3. Telling the father that it is nothing he has done and sharing the nurse's observations of the child 4. Telling the father not to be concerned and stressing that the child will outgrow this developmental phase

3. Telling the father that it is nothing he has done and sharing the nurse's observations of the child

Which nursing actions reflect Leininger's caring theory in practice? 1. The nurse supports and accepts the client's feelings. 2. The nurse promotes beauty, comfort, dignity, and peace. 3. The nurse learns culturally specific behaviors to meet the client's needs. 4. The nurse believes that every caring process has subparts that should be considered while making an effective strategy.

3. The nurse learns culturally specific behaviors to meet the client's needs.

A client tells the nurse, "A man is speaking to me from the corner of the room. Can you hear him?" How should the nurse respond? 1. "What's he saying to you? Does it make any sense?" 2. "Yes, I hear him, but I can't understand what he's saying." 3. "I don't hear him. There's no one in the corner of the room." 4. "No, I don't hear him, but is it making you uncomfortable to hear him?"

4. "No, I don't hear him, but is it making you uncomfortable to hear him?"

A nurse assesses a client who had a gastric resection. During the first 24 hours after surgery, what symptom should the nurse expect to identify? 1. Vomiting 2. Gastric distention 3. Intermittent periods of diarrhea 4. Bloody nasogastric drainage

4. Bloody nasogastric drainage

The mental health nurse is facilitating a therapy group. How can the nurse further develop trust among the members of the group? 1. By discussing the importance of their trusting one another 2. By revealing some personal data as an example of trusting behavior 3. By having group members reveal some personal information about themselves 4. By reminding group members about the need for confidentiality within the group

4. By reminding group members about the need for confidentiality within the group

A school nurse is teaching high school girls regarding the importance of immunizations. Which newborn anomaly can occur if rubella is contracted during the first trimester of pregnancy? 1. Limb abnormalities 2. Hydrocephalus 3. Down syndrome 4. Cardiac anomalies

4. Cardiac anomalies

Which finding indicates that a newborn has vernix caseosa? 1. Brown hair on the skin 2. Rosy to yellowish skin 3. Light-pink to reddish-brown skin 4. Cheese-like substance on the skin

4. Cheese-like substance on the skin

The nurse is caring for a client who underwent intestinal surgery 3 days ago and notices brownish pus with a fecal odor draining from the incision. What should the nurse infer from this finding? 1. Colonization with Proteus 2. Colonization with Pseudomonas 3. Colonization with Staphylococcus 4. Colonization with aerobic coliform and Bacteroides

4. Colonization with aerobic coliform and Bacteroides

Place the pathophysiologic process of tuberculosis infection in its correct order. 1. Necrotic areas calcify or liquefy. 2. Caseation necrosis occurs in the center of the lesion. 3. Areas of caseation undergo resorption, degeneration, and fibrosis. 4. Granulomatous inflammation is created by tuberculosis bacillus in lungs. 5. Granulomatous inflammation becomes surrounded by collagen, fibroblasts, and lymphocytes.

4. Granulomatous inflammation is created by tuberculosis bacillus in lungs. 5. Granulomatous inflammation becomes surrounded by collagen, fibroblasts, and lymphocytes. 2. Caseation necrosis occurs in the center of the lesion. 3. Areas of caseation undergo resorption, degeneration, and fibrosis. 1. Necrotic areas calcify or liquefy.

Which type of hypersensitivity reaction is present in a client with a body temperature of 102 °F, severe joint pain, rashes on the extremities, and enlarged lymph nodes from serum sickness? 1. Delayed reaction 2. Cytotoxic reaction 3. Immediate reaction 4. Immune complex-mediated reaction

4. Immune complex-mediated reaction

A nurse is caring for a client with the diagnosis of bulimia nervosa. What does the nurse understand to be the function of food for individuals with bulimia? 1. Gain attention 2. Control others 3. Avoid growing up 4. Meet emotional needs

4. Meet emotional needs

A 16-year-old client has a blood pressure reading of 119/75. What is the approximate pulse pressure? Record your answer using a whole number. __________ mm Hg

44

A client who has missed two menstrual periods tells a nurse at the prenatal clinic that the home pregnancy test was positive. Her last menstrual period began on June 18. According to Nägele's rule, what is the estimated date of birth (EDB)? 1. March 8 2. March 11 3. March 1 4. March 25

4. March 25

The registered nurse is teaching a nursing student about ways to minimize heat radiation. Which statements made by the nursing student indicate effective learning? Select all that apply. 1. "I will apply an ice pack to the client." 2. "I will cover the client with dark clothes." 3. "I will instruct the client to remove extra clothes." 4. "I will instruct the client to lie in the fetal position." 5. "I will advise the client to wear sparsely woven clothes."

1. "I will apply an ice pack to the client." 2. "I will cover the client with dark clothes." 4. "I will instruct the client to lie in the fetal position."

The nurse finds a client with schizophrenia lying under a bench in the hall. The client says, "God told me to lie here." What is the best response by the nurse? 1 "I didn't hear anyone talking; come with me to your room." 2 "What you heard was in your head; it was your imagination." 3 "Come to the dayroom and watch television; you'll feel better." 4 "God wouldn't tell you to lie there in the hall. God wants you to behave reasonably."

1 "I didn't hear anyone talking; come with me to your room."

The nurse is caring for a client who reports excessive tearing. Which disorders does the nurse suspect could be responsible for the client's condition? Select all that apply. 1. Chalazion 2. Entropion 3. Hordeolum 4. Conjunctivitis 5. Keratoconjunctivitis sicca

1. Chalazion 2. Entropion 4. Conjunctivitis

A nurse is caring for a client who was diagnosed with a myocardial infarction. While caring for the client 2 days after the event, the nurse identifies that the client's temperature is elevated. The nurse concludes that this increase in temperature is most likely the result of what? 1. Tissue necrosis 2. Venous thrombosis 3. Pulmonary infarction 4. Respiratory infection

1. Tissue necrosis

The mother of an infant with Down syndrome asks the nurse what causes the disorder. Before responding, the nurse recalls that the genetic factor of Down syndrome results from what? 1. An intrauterine infection 2. An X-linked genetic disorder 3. Extra chromosomal material 4. An autosomal recessive gene

3. Extra chromosomal material

The nurse educates a client on decreasing the risk of developing antibiotic-resistant infections. Which statement made by the nurse will be most significant? 1. "Wash your hands frequently." 2. "Do not skip any dose of your antibiotics." 3. "Save the unfinished antibiotics for later use." 4. "Stop taking the antibiotics when you feel better."

2. "Do not skip any dose of your antibiotics."

An older adult, accompanied by family members, is admitted to a long-term care facility with symptoms of dementia. What initial statement by the nurse during the admission procedure would be most helpful to this client? 1. "You're a little disoriented now, but don't worry. You'll be all right in a few days." 2. "Don't be afraid. I'm your nurse, and everyone here in the hospital is here to help you." 3. "I'm the nurse on duty today. You're in the hospital. Your family can stay with you for a while." 4. "Let me introduce you to the staff here first. In a little while I'll get you acquainted with our unit routine."

2. "Don't be afraid. I'm your nurse, and everyone here in the hospital is here to help you."

A 56-year-old man is admitted to the inpatient unit after family members report that he seems to be experiencing auditory hallucinations. The man has a history of schizophrenia and has had several previous admissions. Which statement indicates to the nurse that the client is experiencing auditory hallucinations? 1. "Get these horrible snakes out of my room!" 2. "I am not the devil! Stop calling me those names!" 3. "The food on this plate has poison in it, so take it away—I won't eat it." 4. "I did see an alien spaceship last night outside in my yard, and I've felt worse ever since."

2. "I am not the devil! Stop calling me those names!"

A plan of care is created for a term small-for-gestational-age (SGA) neonate who has been admitted to the neonatal intensive care unit (NICU). The goal is for the newborn to reach 5 lb (2300 g) by a specified date. On the specified date the infant weighs 4 lb 2 oz (1871 g). What should the next step be in care planning for this infant? 1. Increase the daily number of calories 2. Change the goal to a more realistic number 3. Evaluate the problem before altering the plan 4. Postpone the evaluation date for another month

3. Evaluate the problem before altering the plan

A nurse is obtaining a health history from a client who is known to be verbally abusive. The client tells the nurse, "You're ugly, and you're probably stupid, too. Why am I stuck with you as my nurse?" What is the best response by the nurse? 1. "It doesn't matter what you think, because I know I'm a capable nurse." 2. "Tell me more about why my caring for you today is so upsetting to you." 3. "If you like, I will arrange to switch assignments so you can have another nurse." 4. "You are talking inappropriately, so I'm going to leave and will come back when you stop being verbally abusive."

4. "You are talking inappropriately, so I'm going to leave and will come back when you stop being verbally abusive."

A woman who is frequently physically abused tells the nurse in the emergency department that it is her fault that her husband beats her. What is the most therapeutic response by the nurse? 1. "Maybe it was your husband's fault, too." 2. "I can't agree with that—no one should be beaten." 3. "Tell me why you believe that you deserve to be beaten." 4. "You say that it was your fault—help me understand that."

4. "You say that it was your fault—help me understand that."

The laboratory international normalized ratio (INR) results of a client receiving warfarin have been variable. The nurse interviews the client to determine factors contributing to the problem. Which is most important for the nurse to identify? 1. Use of analgesics 2. Serum glucose level 3. Serum potassium levels 4. Adherence to the prescribed drug regimen

4. Adherence to the prescribed drug regimen

The nurse is caring for a client who had a thyroidectomy. Which symptoms will the client exhibit if having a thyrotoxic crisis? 1. An increased pulse deficit 2. A decreased blood pressure 3. A decreased heart rate and respirations 4. An increased temperature and pulse rate

4. An increased temperature and pulse rate

A nurse obtains the history of a client with early colon cancer. Which clinical finding does the nurse consider consistent with a diagnosis of cancer of the descending, rather than the ascending, colon? 1. Pain 2. Fatigue 3. Anemia 4. Obstruction

4. Obstruction

Which criteria in a client must be assessed in order to diagnose premenstrual dysphoric disorder (PMDD)? Select all that apply. 1. Symptom-free period in the luteal phase 2. Symptoms that resolve within a few days of menses onset 3. Symptoms caused by the exacerbations of other disorder 4. Symptoms such as irritability, anxiety, or depressed mood 5. Symptoms that interfere with work or interpersonal relationships

4. Symptoms such as irritability, anxiety, or depressed mood 5. Symptoms that interfere with work or interpersonal relationships

Which nursing actions are accurate when measuring a head circumference for an infant? Select all that apply. 1. Using a cloth tape for accuracy 2. Obtaining one measurement per visit 3. Documenting the information in the progress notes 4. Using paper tape marked with tenths of a centimeter 5. Placing the tape slightly above the eyebrows and pinna of the ears

4. Using paper tape marked with tenths of a centimeter 5. Placing the tape slightly above the eyebrows and pinna of the ears

A nurse is working with a client experiencing a major depressive episode. What is a long-term outcome for this client? 1. Talking openly about the depressed feelings 2. Identifying and using new defense mechanisms 3. Discussing the unconscious source of the anger 4. Verbalizing realistic perceptions of self and others

4. Verbalizing realistic perceptions of self and others

A client has returned from surgery with a nephrostomy tube. Which is the most essential nursing intervention for this client? 1. Ensure free drainage of urine. 2. Milk the tube every 2 hours. 3. Keep an accurate record of intake and output. 4. Instill 12 mL of normal saline every 8 hours

1. Ensure free drainage of urine.

A client newly diagnosed with scleroderma states, "Where did I get this from?" How should the nurse reply? 1. "The exact cause is unknown, but it is thought to be a result of autoimmunity." 2. "The exact cause is unknown, but it is thought to be a result of ocular motility." 3. "The exact cause is unknown, but it is thought to be a result of increased amino acid metabolism." 4. "The exact cause is unknown, but it is thought to be a result of defective sebaceous gland formation."

1. "The exact cause is unknown, but it is thought to be a result of autoimmunity."

A client undergoes dilation and curettage (D & C) after an early miscarriage (spontaneous abortion). The nurse finds her crying later in the day. What is the most appropriate statement by the nurse at this time? 1. "This must be a very difficult experience for you to deal with." 2. "You'll have other children to take the place of the one you lost." 3. "Of course you're sad now, but at least you know you can get pregnant." 4. "I know how you feel, but when a woman miscarries, it's usually for the best."

1. "This must be a very difficult experience for you to deal with."

A nurse educates parents about how to communicate with their 14-year-old. Which statement should the nurse make? 1. "You should ask your child closed-ended questions." 2. "You should avoid involving other individuals and resources." 3. "You should avoid discussing sensitive issues with your child." 4. "You should look for the meaning behind your child's words or actions."

1. "You should ask your child closed-ended questions."

The nurse is performing physical assessments for children in a daycare center. Which children should require a head circumference in order to monitor growth patterns? Select all that apply. 1. A 6-month-old infant who is breastfed 2. A 15-month-old toddler who has asthma 3. A 3-year-old child whose birthday was the day prior 4. A 5-year-old who will attend kindergarten in the fall 5. An 8-year-old child who will begin playing soccer next week

1. A 6-month-old infant who is breastfed 2. A 15-month-old toddler who has asthma 3. A 3-year-old child whose birthday was the day prior

After multiple upper respiratory infections, a school-aged child undergoes a tonsillectomy and adenoidectomy. Two weeks after surgery the nurse assesses the child's condition. On what should the nurse focus? Select all that apply. 1. Taste 2. Smell 3. Hearing 4. Breathing 5. Facial symmetry

1. Taste 2. Smell 3. Hearing 4. Breathing

When assessing an 85-year-old client's vital signs, the nurse anticipates a number of changes in cardiac output that result from the aging process. Which finding is consistent with a pathologic condition rather than the aging process? 1. A pulse rate irregularity 2. Equal apical and radial pulse rates 3. A pulse rate of 60 beats per minute 4. An apical rate obtainable at the fifth intercostal space and midclavicular line

1. A pulse rate irregularity

A diabetic client survives a fire. The client has a head injury, tachycardia, and pale and ashen-colored skin. Which primary interventions should the nurse perform? Select all that apply. 1. Administering cool intravenous fluids 2. Placing the client in a cool environment 3. Managing and maintaining airway, breathing, and circulation 4. Monitoring heart rhythm, oxygen saturation, and urine output 5. Monitoring temperature, vital signs, and level of consciousness

1. Administering cool intravenous fluids 2. Placing the client in a cool environment 3. Managing and maintaining airway, breathing, and circulation

A nurse is reviewing the history, physical examination, and diagnostic test results of a client with colitis. What clinical findings are associated with this disorder? Select all that apply. 1. Anemia 2. Diarrhea 3. Hemoptysis 4. Abdominal cramps 5. Decreased white blood cells

1. Anemia 2. Diarrhea 4. Abdominal cramps

Which clinical findings are observed in a client suffering from an imbalance of adrenocorticotropic hormone? Select all that apply. 1. Anorexia 2. Hyponatremia 3. Slowed cognition 4. Postural hypotension 5. Decreased muscle strength

1. Anorexia 2. Hyponatremia 4. Postural hypotension

A client is admitted for repair of bilateral inguinal hernias. Before surgery the nurse assesses the client for indicators that strangulation of the intestine may have occurred. What is an early indicator of strangulation? 1. Increased flatus 2. Projectile vomiting 3. Sharp abdominal pain 4. Decreased bowel sounds

3. Sharp abdominal pain

The nurse is providing postoperative care to a client on the second day after the client had a coronary artery bypass surgery. When assessing the water-seal chamber of the chest drainage device, the nurse observes that the fluid no longer fluctuates. What should the nurse do? 1. Assess for obstructions in the chest tube 2. Increase the amount of continuous suction 3. Add sterile water to the water-seal chamber 4. Make preparations to remove the chest tube

1. Assess for obstructions in the chest tube

A client presents to the emergency department with weakness, dizziness, and difficulty breathing. The nurse performs an electrocardiogram (ECG) and notices this arrhythmia. Which arrhythmia is the client exhibiting? 1. Atrial fibrillation (AF) 2. Ventricular tachycardia (VT) 3. Junctional tachycardia 4. Supraventricular tachycardia (SVT)

1. Atrial fibrillation (AF)

The nurse provides nutritional counseling to the parents of a 6-month-old formula-fed infant who will begin eating solid foods. Which statement by a parent indicates understanding of the nurse's advice? 1. "I'll keep giving him formula instead of regular cow's milk." 2. "I'll buy plenty of pureed spinach so she gets enough iron." 3. "Using a natural sweetener like honey is better than using table sugar." 4. "Baby food is sterilized, so it's better to feed directly from the jar than from a bowl."

1. "I'll keep giving him formula instead of regular cow's milk."

A registered nurse is teaching a nursing student about Piaget's theory of cognitive development. What information should the nurse provide about the sensorimotor period? Select all that apply. 1. "In this stage, the child learns that he or she is separate from his or her parents or favorite toy." 2. "In this stage, the child develops a schema or action pattern for dealing with his or her environment." 3. "In this stage, the child believes that non-living objects have realistic thoughts, wishes, and feelings." 4. "In this stage, the child learns about himself or herself and the environment through motor and reflex actions." 5. "In this stage, the child promotes his or her cognitive development and learns about the world through playing."

1. "In this stage, the child learns that he or she is separate from his or her parents or favorite toy." 2. "In this stage, the child develops a schema or action pattern for dealing with his or her environment." 4. "In this stage, the child learns about himself or herself and the environment through motor and reflex actions."

What statements about culturally congruent care by the student nurse are correct? Select all that apply 1. "It is the main goal of transcultural nursing." 2. "It is provided through cultural competence." 3. "It is provided in accordance with set criteria." 4. "It is bound to the professional health care system." 5. "It depends on the patterns and needs of an individual."

1. "It is the main goal of transcultural nursing." 2. "It is provided through cultural competence." 5. "It depends on the patterns and needs of an individual."

Which parental statement indicates the need for further education regarding the psychosocial development that occurs during infancy? 1. "My older kids are so excited that our 10-month-old can play hide-and-seek with them." 2. "Peek-a-boo is an appropriate activity to initiate with my baby around 9 months of age." 3. "I just bought my 6-month-old some new rattles to play with because they are easy to grasp." 4. "It is important that my baby develops trust so we always respond when he cries for us at night."

1. "My older kids are so excited that our 10-month-old can play hide-and-seek with them."

A client who was forced into early retirement is found to have severe depression. The client says, "I feel useless, and I've got nothing to do." What is the best initial response by the nurse? 1. "Tell me more about feeling useless." 2. "Volunteering can help you fill your time." 3. "Your illness is adding to your current feelings." 4. "Let's talk about what you'd like to be doing right now."

1. "Tell me more about feeling useless."

A client with mild preeclampsia is instructed to rest at home. She asks the nurse, "What do you mean by rest?" What is the most appropriate response? 1. "Tell me what you consider rest." 2. "Take three or four naps a day." 3. "Stay off your feet as much as possible." 4. "Would you like to know what I think it means?"

1. "Tell me what you consider rest."

What would the nurse describe as a similarity between the growth and development of toddlers and preschoolers? 1. Both gain 5 to 7 pounds per year. 2. Both need an equal amount of calories. 3. Both grow about 2.5 inches in height per year. 4. Both need at least 12 hours of sleep every night.

1. Both gain 5 to 7 pounds per year.

How do adolescents establish family identity during psychosocial development? Select all that apply. 1. By acting independently to make his or her own decisions 2. By evaluating his or her own health with a feeling of well-being 3. By fostering his or her own development within a balanced family structure 4. By building close peer relationships to achieve acceptance in the society 5. By achieving marked physical changes

1. By acting independently to make his or her own decisions 3. By fostering his or her own development within a balanced family structure

Which criteria should a nurse assess before preparing to discharge a pediatric client after the administration of conscious or moderate sedation? Select all that apply. 1. Check if gag reflexes are intact 2. Check if pain and ambulation are at a base level 3. Check if a reversal drug is administered just before discharge 4. Check if the oxygen saturation is below 95% on room air half an hour after the administration of the last dose 5. Check if an adult is accompanying the child and can remain with him or her for at least two half-lives of the anesthetic drug

1. Check if gag reflexes are intact 2. Check if pain and ambulation are at a base level 5. Check if an adult is accompanying the child and can remain with him or her for at least two half-lives of the anesthetic drug

Which clients should be considered for assessing the carotid pulse? Select all that apply. 1. Client with cardiac arrest 2. Client indicated for Allen test 3. Client under physiologic shock 4. Client with impaired circulation to foot 5. Client with impaired circulation to hand

1. Client with cardiac arrest 3. Client under physiologic shock

A nurse is caring for a client who has had multiple myocardial infarctions and has now developed cardiogenic shock. Which clinical manifestation supports this diagnosis? 1. Cold, clammy skin 2. Slow, bounding pulse 3. Increased blood pressure 4. Hyperactive bowel sounds

1. Cold, clammy skin

A 2-year-old child is admitted to the pediatric unit with a diagnosis of thalassemia major (Cooley anemia). The parents are told that there is no cure, but the anemia can be treated with frequent blood transfusions. The father tells the nurse he is glad that there is a treatment that "fixes" his child's problem. Before responding, the nurse should recall that blood transfusions do what? 1. Correct the anemia, but may cause other problems 2. Reverse the anemia, but also present a risk of hepatitis 3. Are a supportive treatment; fewer will be needed as the child grows older 4. Are a replacement for defective red blood cells; they are like giving insulin to a person with diabetes

1. Correct the anemia, but may cause other problems

A client with mild preeclampsia is admitted to the high-risk prenatal unit because of a progressive increase in her blood pressure. The nurse reviews the primary healthcare provider's prescriptions. Which prescriptions does the nurse expect to receive for this client? Select all that apply. 1. Daily weight 2. Side-lying bed rest 3. 2 g/day sodium diet 4. Deep tendon reflexes 5. Glucose tolerance test

1. Daily weight 2. Side-lying bed rest 4. Deep tendon reflexes

A client developed acute herpes zoster and was treated with antiviral medication within 72 hours of the appearance of the rash. The client is reporting persistent pain 1 week later. What does the nurse identify as the cause of the posttherapeutic neuralgia? 1. Damage to the nerves 2. Untreated major depression 3. Scarring in the area of the rash 4. Continued presence of the skin rash

1. Damage to the nerves

Which symptoms indicate to the nurse that the client has an inadequate fluid volume? Select all that apply. 1. Decreased urine 2. Hypotension 3. Dyspnea 4. Dry mucous membranes 5. Pulmonary edema 6. Poor skin turgor

1. Decreased urine 2. Hypotension 4. Dry mucous membranes 6. Poor skin turgor

A female client reports excessive hair growth on the face and chest. The nurse suspects ovarian dysfunction. Which findings support this assessment? Select all that apply. 1. Deepened voice 2. Enlarged clitoris 3. Capillary fragility 4. Changes in fat distribution 5. Increased thyroid gland activity

1. Deepened voice 2. Enlarged clitoris

An infant is born with a bilateral cleft palate. Plans are made to begin reconstruction immediately. What nursing intervention should be included to promote parent-infant attachment? 1. Demonstrating positive acceptance of the infant 2. Placing the infant in a nursery away from view of the general public 3. Explaining to the parents that the infant will look normal after the surgery 4. Encouraging the parents to limit contact with the infant until after the surgery

1. Demonstrating positive acceptance of the infant

A client with acquired immunodeficiency syndrome (AIDS) is receiving a treatment protocol that includes a protease inhibitor. When assessing the client's response to this drug, which common side effect should the nurse expect? 1. Diarrhea 2. Hypoglycemia 3. Paresthesias of the extremities 4. Seeing yellow halos around lights

1. Diarrhea

The mother of an 11-month-old infant reports that the baby has allergies. After an assessment, the primary healthcare provider also suspects anemia. Which questions would the primary healthcare provider most likely ask the mother? Select all that apply. 1. Do you use 2% cow's milk? 2. Do you breastfeed? 3. Do you use whole cow's milk? 4. Do you use alternate milk products? 5. Do you provide 18 to 21 ounces of breast milk per day?

1. Do you use 2% cow's milk? 3. Do you use whole cow's milk? 4. Do you use alternate milk products?

A client at 36 hours' postpartum is being treated with subcutaneous enoxaparin for deep vein thrombosis of the left calf. Which client adaptation is of most concern to the nurse? 1. Dyspnea 2. Pulse rate of 62 beats/min 3. Blood pressure of 136/88 mm Hg 4. Homan sign in the left leg

1. Dyspnea

Which statements about acne in adolescents are true? Select all that apply. 1. Early acne occurs in the midface region. 2. Acne is more common in girls than boys. 3. Acne usually occurs in middle to late adolescence. 4. Intake of dairy products can contribute to acne severity. 5. Acneiform eruptions are predominant in young children

1. Early acne occurs in the midface region. 3. Acne usually occurs in middle to late adolescence. 4. Intake of dairy products can contribute to acne severity.

Which type of burn/injury may cause a client to have a cervical spine injury? 1. Electrical burns 2. Chemical burns 3. Inhalation injury 4. Cold thermal injury

1. Electrical burns

While caring for a pregnant client with a body mass index of 32 during labor, the nurse observes that the second stage of labor lasts for about 11 minutes. The nurse also finds that the expected birth weight of the fetus is around 4200 g. Which complication does the nurse anticipate in the neonate after birth? 1. Erb palsy (Erb- Duchenne paralysis) 2. Klumpke palsy 3. Strawberry hemangioma 4. Erythema toxicum neonatorum

1. Erb palsy (Erb- Duchenne paralysis)

A nurse assesses a newly admitted client with a diagnosis of pulmonary tuberculosis (TB). Which clinical findings support this diagnosis? Select all that apply. 1. Fatigue 2. Polyphagia 3. Hemoptysis 4. Night sweats 5. Black tongue

1. Fatigue 3. Hemoptysis 4. Night sweats

A preterm newborn is admitted to the neonatal intensive care unit (NICU). Which concern is most commonly expressed by NICU parents? 1. Fear of handling the infant 2. Delayed ability to bond with the infant 3. Prolonged hospital stay needed by the infant 4. Inability to provide breast milk for the infant

1. Fear of handling the infant

A newborn experiences a hypothermic period while being bathed and having clothing changed. Once the hypothermic episode has been identified and treated, what is the next nursing action? 1. Feeding the infant 2. Requesting a complete blood count 3. Monitoring the infant for hyperthermia 4. Allowing the infant to rest undisturbed

1. Feeding the infant

A client is admitted to the hospital for surgery for rectosigmoid colon cancer, and the nurse is obtaining a health history as part of the admission process. What clinical findings associated with rectosigmoid colon cancer does the nurse expect the client to report? Select all that apply. 1. Feeling tired 2. Rectal bleeding 3. Inability to digest fat 4. Change in the shape of stools 5. Feeling of abdominal bloating

1. Feeling tired 2. Rectal bleeding 4. Change in the shape of stools 5. Feeling of abdominal bloating

The nurse suspects the Jarisch-Herxheimer reaction in a client with syphilis who is on antibiotic therapy. Which symptoms in the client support the nurse's suspicion? Select all that apply. 1. Fever 2. Hypertension 3. Vasoconstriction 4. Generalized ache 5. Pain at the injection site

1. Fever 4. Generalized ache 5. Pain at the injection site

A client who had surgery for a ruptured appendix develops peritonitis. Which clinical findings related to peritonitis should the nurse expect the client to exhibit? Select all that apply. 1. Fever 2. Hyperactivity 3. Extreme hunger 4. Urinary retention 5. Abdominal muscle rigidity

1. Fever 5. Abdominal muscle rigidity

The nurse is assessing an older adult client with suspected hearing loss. Which observations made by the nurse in the client indicates a decrease in hearing acuity? Select all that apply. 1. Frequent usage of words such as "what" 2. Postural changes while listening to the speaker 3. Bending towards the other person while talking 4. Mismatch in the questions asked and the responses given 5. Startled expression when there is any unexpected sound in the environment

1. Frequent usage of words such as "what" 2. Postural changes while listening to the speaker 3. Bending towards the other person while talking 4. Mismatch in the questions asked and the responses given

What is an important aspect of nursing care for a client exhibiting psychotic patterns of thinking and behavior? 1. Helping keep the client oriented to reality 2. Involving the client in activities throughout the day 3. Helping the client understand that it is harmful to withdraw from situations 4. Encouraging the client to discuss why interacting with other people is being avoided

1. Helping keep the client oriented to reality

A nurse is teaching the parents of a school-aged child with sickle cell anemia about ways to prevent sickling. What should the nurse explain as the primary cause of sickling? 1. Hypoxia 2. Hemodilution 3. Hypocalcemia 4. Hemoglobin

1. Hypoxia

A nurse is teaching Hands Only Basic Life Support for adults in the community. What should the rescuer do first after determining that the person is not responding, and the emergency medical system has been activated? 1. Identify the absence of pulse. 2. Give two rescue breaths with a CPR mask. 3. Perform the head tilt-chin lift maneuver. 4. Perform chest compression at a rate of 100/min

1. Identify the absence of pulse.

A nurse is caring for a client who experienced a crushing chest injury. A chest tube is inserted. Which observation indicates a desired response to this treatment? 1. Increased breath sounds 2. Increased respiratory rate 3. Crepitus detected on palpation of the chest 4. Constant bubbling in the drainage collection chamber

1. Increased breath sounds

A client's membranes ruptured 20 hours before admission. The client was in labor for 24 hours before giving birth. For which postpartum complication is this client at risk? 1. Infection 2. Hemorrhage 3. Uterine atony 4. Amniotic fluid embolism

1. Infection

A client reports fever, headache, extreme tiredness, dry cough, sore throat, runny nose, muscle aches, nausea, vomiting, and diarrhea. Which organism is responsible for this condition? 1. Influenza virus 2. Toxoplasma gondii 3. Human herpes virus-8 4. Cryptosporidium muris

1. Influenza virus

An adolescent is found to have type 1 diabetes. The nurse plans to teach the adolescent that dietary control and exercise can help regulate the disorder. What additional information should the nurse include in the teaching plan? Select all that apply. 1. Insulin therapy 2. Prophylactic antibiotics 3. Blood glucose monitoring 4. Oral hypoglycemic agents 5. Adherence to the treatment regimen

1. Insulin therapy 3. Blood glucose monitoring 5. Adherence to the treatment regimen

After counseling an older widowed client, a nurse concludes that the grieving process has been successfully completed when the client does what? 1. Is able to plan to start new relationships 2. Talks about the deceased spouse at great length 3. Ignores the deceased spouse's less-than-perfect qualities 4. Decides to leave the deceased spouse's study as it was before the death

1. Is able to plan to start new relationship

The nurse is caring for a client with a body surface burn injury of 55%. Which information will the nurse consider when planning care for this client? 1. Is prone to poor healing because of a hypermetabolic state 2. Has a decreased risk of infection when in a hypermetabolic state 3. Needs a cool environment to decrease caloric need 4. Will need 20 calories/kg during the healing process

1. Is prone to poor healing because of a hypermetabolic state

A nursing instructor asks a nursing student to explain the teaching methods to be used for adolescents. Which statement by the student indicates a need for further teaching? Select all that apply. 1. Keep teaching sessions short 2. Use teaching as a collaborative activity 3. Use problem-solving to help adolescents make choices 4. Encourage them to learn together, using pictures and short stories 5. Help adolescents learn about feelings and the need for self-expression

1. Keep teaching sessions short 4. Encourage them to learn together, using pictures and short stories

A nurse performs Leopold maneuvers on a newly admitted client in labor. Palpation reveals a soft, firm mass in the fundus; a firm, smooth mass on the mother's left side; several knobs and protrusions on the mother's right side; and a hard, round, movable mass in the pubic area with the brow on the right. On the basis of these findings, the nurse determines that the fetal position is what? 1. LOA (left occiput anterior) 2. ROA (right occiput anterior) 3. LMP (left mentum posterior) 4. RMP (right mentum posterior)

1. LOA (left occiput anterior)

During the progressive stage of shock, anaerobic metabolism occurs. Which complication should the nurse anticipate in this client? 1. Metabolic acidosis 2. Metabolic alkalosis 3. Respiratory acidosis 4. Respiratory alkalosis

1. Metabolic acidosis

Which data should the nurse anticipate when conducting a developmental assessment for a 5-year-old client? Select all that apply. 1. Names coins correctly 2. Has a vocabulary of 1500 words 3. Participates in parallel play 4. Ties shoe laces independently 5. Has hand dominance established

1. Names coins correctly 4. Ties shoe laces independently 5. Has hand dominance established

Which stages would the nurse explain that a toddler goes through, according to Freud's theory? Select all that apply. 1. Oral 2. Anal 3. Phallic 4. Genital 5. Latency

1. Oral 2. Anal

A pregnant client is prescribed heparin to prevent the risk of thromboembolism. Which adverse effects should the nurse anticipate with this medication? Select all that apply. 1. Osteoporosis 2. Suppress contractions in labor 3. Increased risk of serious bleeding 4. Stimulation of uterine contraction 5. Compression fractures of the spine

1. Osteoporosis 5. Compression fractures of the spine

The parents of a 12-year-old boy with cystic fibrosis (CF) ask the nurse why he needs a glucose tolerance test. What information should the nurse consider before replying? 1. Pancreatic scarring predisposes the child to diabetes. 2. The thickened mucus blocks the insulin-secreting glands. 3. The test reveals the degree to which the child adheres to the diet. 4. Adjustments of the dosage of pancreatic enzymes are based on the results of the test.

1. Pancreatic scarring predisposes the child to diabetes.

What safety measure should the nurse instruct parents to follow when their child is a toddler? 1. Place window guards on all windows. 2. Have the toddler sleep on the back or side. 3. Start swimming training for the toddler under supervision. 4. Teach the child how to cross streets and walk in parking lots.

1. Place window guards on all windows.

The nurse accompanies a 3-year-old child to the playroom. The toddler seems afraid to select a toy or activity. What age-appropriate play materials should the nurse offer? Select all that apply. 1. Plastic tea set 2. Mold and clay 3. Play telephone 4. Pencil and paper 5. Simple video game

1. Plastic tea set 2. Mold and clay 3. Play telephone

A nurse is assessing a client admitted to the hospital with a tentative diagnosis of a pituitary tumor. What signs of Cushing syndrome does the nurse identify? 1. Retention of sodium and water 2. Hypotension and a rapid, thready pulse 3. Increased fatty deposition in the extremities 4. Hypoglycemic episodes in the early morning

1. Retention of sodium and water

While assessing a client with acquired immunodeficiency syndrome (AIDS), the nurse suspects that the client has developed cryptococcosis. Which clinical manifestations support the nurse's suspicion of a cryptococcosis infection? Select all that apply. 1. Seizures 2. Dyspnea 3. Blurred vision 4. Neurologic deficits 5. Enlarged lymph nodes

1. Seizures 3. Blurred vision 4. Neurologic deficits

The nurse is caring for a 1-hour-old newborn. Which assessment characteristics represent a preterm gestational age? 1. Skin: thin, veins visible; breasts: flat areolae, no buds; plantar creases: absent; lanugo: abundant 2. Skin: parchment/wrinkled; breasts: flat areolae, no buds; plantar creases: cover entire sole; lanugo: absent 3. Skin: thin, veins visible; breasts: flat areolae, no buds; plantar creases: covering the entire sole; lanugo: abundant 4. Skin: cracking/few veins; breasts: raised areolae (3- to 4-mm buds); plantar creases: covering the anterior two thirds of the sole; lanugo: thinning

1. Skin: thin, veins visible; breasts: flat areolae, no buds; plantar creases: absent; lanugo: abundant

An older client experiences a cerebral vascular accident (CVA) and has right-sided hemiplegia and expressive aphasia. The client's children ask the nurse which functions will be impaired. Which abilities does the nurse explain will be affected? 1. Stating wishes verbally 2. Recognizing familiar objects 3. Comprehending written words 4. Understanding verbal communication

1. Stating wishes verbally

A client has laparoscopic surgery to remove a calculus from the common bile duct. What postoperative client response indicates to the nurse that bile flow into the duodenum is reestablished? 1. Stools become brown 2. Liver tenderness is relieved 3. Colic is absent after ingestion of fats 4. Serum bilirubin level returns to the expected range

1. Stools become brown

A client is admitted to the hospital with a diagnosis of lower extremity arterial disease (LEAD) or peripheral arterial disease. Which is the most beneficial lifestyle modification the nurse should teach this client? 1. Stop smoking 2. Control blood glucose 3. Start a walking program 4. Eat a low-fat, low-cholesterol diet

1. Stop smoking

On reviewing the data of a client with thyroid disorder, the primary healthcare provider prescribed atenolol. Which assessment findings would indicate the need for atenolol therapy? Select all that apply. 1. Tachycardia 2. Atrial fibrillation 3. Distant heart sounds 4. Systolic hypertension 5. Decreased cardiac output

1. Tachycardia 2. Atrial fibrillation 4. Systolic hypertension

What does a community-based nurse do as a change agent? Select all that apply. 1. The nurse empowers clients and their families to creatively solve problems. 2. The nurse works with clients to solve problems and helps clients identify an alternative care facility. 3. The nurse helps clients gain the skills and knowledge needed to provide self-care. 4. The nurse empowers clients to become instrumental in creating change within a health care agency. 5. The nurse does not make decisions but rather helps clients reach decisions that are best for them.

1. The nurse empowers clients and their families to creatively solve problems. 2. The nurse works with clients to solve problems and helps clients identify an alternative care facility. 4. The nurse empowers clients to become instrumental in creating change within a health care agency.

A newborn is being treated with phototherapy for hyperbilirubinemia. What is the nurse's role when providing phototherapy? 1. Turning the infant every 2 hours 2. Measuring the bilirubin level every 2 hours 3. Maintaining the infant on daily 24-hour phototherapy 4. Applying a sterile gauze pad to the infant's umbilical stump

1. Turning the infant every 2 hours

A pregnant client with iron-deficiency anemia is prescribed iron supplements daily. To help the client increase iron absorption, the nurse should suggest that the client eat foods high in which substance? 1. Vitamin C 2. Fat content 3. Water content 4. Vitamin B complex

1. Vitamin C

The nurse caring for a 3-year-old child with meningitis should be alert for which signs and symptoms of increased intracranial pressure? Select all that apply. 1. Vomiting 2. Headache 3. Irritability 4. Tachypnea 5. Hypotension

1. Vomiting 2. Headache 3. Irritability

A nurse notices a firm, edematous, irregularly shaped skin lesion on a client who reports an insect bite. Which skin lesion is this? 1. Wheal 2. Plaque 3. Vesicle 4. Pustule

1. Wheal

A client with bubonic plague has a body temperature of 103° F associated with chills, swollen glands, headache, and weakness. Which microorganism is most likely responsible for the client's condition? 1. Yersinia pestis 2. Bordetella pertusis 3. Mycobacterium tuberculosis 4. Corynebacterium diphtheria

1. Yersinia pestis

A client is admitted to the mental health unit with the diagnosis of major depressive disorder. Which statement alerts the nurse to the possibility of a suicide attempt? 1. "I don't feel too good today." 2. "I feel much better; today is a lovely day." 3. "I feel a little better, but it probably won't last." 4. "I'm really tired today, so I'll take things a little slower."

2. "I feel much better; today is a lovely day."

Which intervention does the nurse implement to develop a caring relationship with the client's family? 1. Deciding healthcare options for the client 2. Identifying the client's family members and their roles 3. Declining to inform the client's family after performing a procedure 4. Refraining from discussing the client's health with the family

2. Identifying the client's family members and their roles

The parents tell the nurse that their preschooler often awakes from sleep screaming in the middle of the night. The preschooler is not easily comforted and screams if the parents try to restrain the child. What does the nurse instruct the parents? 1. "Always read a story to the child before bedtime." 2. "Intervene only if necessary to protect the child from injury." 3. "Discuss counseling options with the primary health care provider." 4. "Try to wake the child and ask the child to describe the dream."

2. "Intervene only if necessary to protect the child from injury."

After a difficult labor a client gives birth to a 9-lb (4 kg) boy who expires shortly afterward. That evening the client tearfully describes to the nurse her projected image of her son and what his future might have been. What is the nurse's most therapeutic response? 1. "I guess you wanted a son very much." 2. "It must be difficult to think of him now." 3. "I'm sure he would have been a wonderful child." 4. "If you dwell on this now, your grief will be harder to bear."

2. "It must be difficult to think of him now."

A 15-year-old client tearfully states that her father has been sexually abusing her for the past 8 years. What statement should the nurse initially respond with? 1. "Which type of incidents preceded the abuse?" 2. "Sharing this information is a positive step in getting help." 3. "I have to report this to child protective services right now." 4. "What kinds of things does he do to you when he abuses you?"

2. "Sharing this information is a positive step in getting help."

A registered nurse is supervising a student nurse while assessing a 70-year-old client who is receiving aminoglycoside therapy. Which statement about the client's condition requires correction? 1. "The client may have deterioration of the cochlea." 2. "The client may have thinning of the tympanic membrane." 3. "The client may have an inability to hear high-frequency sounds." 4. "The client may have an inability to differentiate between consonants."

2. "The client may have thinning of the tympanic membrane."

A depressed client cries when the family does not visit. What is the most therapeutic response by the nurse? 1. "It's difficult to realize that no one cares about you." 2. "Your family didn't visit, and now you're feeling rejected." 3. "It's terrible to have such negative thoughts about yourself." 4. "Your family members work—that's why they don't visit you."

2. "Your family didn't visit, and now you're feeling rejected."

Pharmacokinetic factors determine the concentration of a drug at its site of action. These are different in neonates and infants than they are in small children or adults. Arrange the factors in the order in which they occur. 1. Excretion 2. Absorption 3. Distribution 4.Metabolism

2. Absorption 3. Distribution 4.Metabolism 1. Excretion

When the fetal head begins to crown during an emergency precipitous birth, how should the nurse respond? 1. Pressing firmly on the fundus 2. Applying gentle perineal pressure 3. Encouraging the client to push forcefully 4. Telling the client to take prolonged deep breaths

2. Applying gentle perineal pressure

A client with chronic obstructive pulmonary disease is admitted to the hospital with a tentative diagnosis of pleuritis. When caring for this client, what should the nurse do? 1. Administer opioids frequently 2. Assess for signs of pneumonia 3. Give medication to suppress coughing 4. Limit fluid intake to prevent pulmonary edema

2. Assess for signs of pneumonia

A client with a traumatic brain injury is demonstrating signs of increasing intracranial pressure, which may exert pressure on the medulla. What should the nurse assess to determine involvement of the medulla? Select all that apply. 1. Taste 2. Breathing 3. Heart rate 4. Fluid balance 5. Voluntary movement

2. Breathing 3. Heart rate

A nurse is caring for a client who is admitted to the hospital with severe dyspnea and a diagnosis of cancer of the lung. What does the nurse conclude is the probable cause of the severe dyspnea? 1. Abdominal distention or pressure 2. Bronchial obstruction or pleural effusion 3. Fluid retention as a result of renal failure 4. Anxiety associated with pain on inspiration

2. Bronchial obstruction or pleural effusion

A client with acute kidney injury states, "Why am I twitching and my fingers and toes tingling?" Which process should the nurse consider when formulating a response to this client? 1. Acidosis 2. Calcium depletion 3. Potassium retention 4. Sodium chloride depletion

2. Calcium depletion

The nurse is preparing a teaching plan for clients receiving antitubercular medications. Which teaching plan needs correction? 1. Clients taking ethambutol should drink plenty of fluids. 2. Clients taking Isoniazid should take the drug with food. 3. Clients taking pyrazinamide should wear a hat while going out in the sun. 4. Clients taking rifampin should use other contraceptive methods even after stopping the medication.

2. Clients taking Isoniazid should take the drug with food.

A 28-year-old woman comes into the clinic and tells the nurse that she fears that she is infertile, because she has been trying to become pregnant unsuccessfully for 2 years. While collecting the health history the nurse learns that the client experiences irregular and infrequent menstrual periods. The client is overweight and has severe acne and alopecia. The primary healthcare provider diagnoses the condition polycystic ovarian syndrome (PCOS). Which of the following interventions is the most important? 1. Consoling the client over her inability to have children 2. Discussing weight loss, exercise, and a balanced low-fat diet 3. Providing information to the client on how to prepare for surgery 4. Informing the client that there are no long-term complications of PCOS

2. Discussing weight loss, exercise, and a balanced low-fat diet

A 6-year-old child with autism is nonverbal and makes limited eye contact. What should the nurse do initially to promote social interaction? 1. Encourage the child to sing songs with the nurse. 2. Engage in parallel play while sitting next to the child. 3. Provide opportunities for the child to play with other children. 4. Use therapeutic holding when the child does not respond to verbal interactions.

2. Engage in parallel play while sitting next to the child.

During a survey, the community nurse meets a client who has not visited a gynecologist after the birth of her second child. The client says that her mother or sister never had annual gynecologic examinations. Which factor is influencing the client's health practice? 1. Spiritual belief 2. Family practices 3. Emotional factors 4. Cultural background

2. Family practices

A child is diagnosed with classic hemophilia. A nurse teaches the child's parents how to administer the plasma component factor VIII through a venous port. It is to be given three times a week. When should the parents administer this therapy? 1. Whenever a bleed is suspected 2. In the morning on scheduled days 3. At bedtime while the child is lying quietly in bed 4. On a regular schedule at the parents' convenience

2. In the morning on scheduled days

The nurse teaching a health awareness class identifies which situation as being the highest risk factor for the development of a deep vein thrombosis (DVT)? 1. Pregnancy 2. Inactivity 3. Aerobic exercise 4. Tight clothing

2. Inactivity

Which nursing action is important when suctioning the secretions of a client with a tracheostomy? 1. Use a new sterile catheter with each insertion. 2. Initiate suction as the catheter is being withdrawn. 3. Insert the catheter until the cough reflex is stimulated. 4. Remove the inner cannula before inserting the suction catheter.

2. Initiate suction as the catheter is being withdrawn.

The nurse is caring for a preschooler diagnosed as suffering from frequent episodes of sleep terrors. Which statements describing the nature of sleep terrors does the nurse know to be true? Select all that apply. 1. It is followed by full waking. 2. It usually occurs 1 to 4 hours after falling asleep. 3. It takes place during rapid-eye movement (REM) sleep. 4. The child rapidly returns to sleep after an episode of sleep terrors. 5. The child is aware of and reassured by another's presence after an episode of sleep terrors.

2. It usually occurs 1 to 4 hours after falling asleep. 4. The child rapidly returns to sleep after an episode of sleep terrors.

A client is at high risk for developing ascites because of cirrhosis of the liver. How should the nurse assess for the presence of ascites? 1. Observe the client for signs of respiratory distress. 2. Percuss the client's abdomen and listen for dull sounds. 3. Palpate the lower extremities over the tibia and observe for edema. 4. Listen for decreased or absent bowel sounds while auscultating the abdomen.

2. Percuss the client's abdomen and listen for dull sounds.

A client at 40 weeks' gestation is admitted to the birthing unit in labor. During the initial examination the nurse uses Leopold maneuvers to palpate the abdomen. The purpose of this intervention is to assess the what? 1. Station of the fetus 2. Position of the fetus 3. Duration of the contractions 4. Frequency of the contractions

2. Position of the fetus

When monitoring fluids and electrolytes, the nurse recalls that the major cation-regulating intracellular osmolarity is what? 1. Sodium 2. Potassium 3. Calcium 4. Calcitonin

2. Potassium

A client arrives at the emergency room complaining of chest pain and dizziness. The client has a history of angina. The primary healthcare provider prescribes an electrocardiogram (ECG) and lab tests. A change in which component of the ECG tracing should the nurse recognize as the client actively having a myocardial infarction (MI)? 1. QRS complex 2. S-T segment 3. P wave 4. R wave

2. S-T segment

A nurse is assessing a newborn with caput succedaneum. How does the nurse explain the cause of this fetal condition to the new mother? 1.Overlap of fetal bones as they pass through the maternal birth canal 2. Swelling of the soft tissue of the scalp as a result of pressure during labor 3. Hemorrhage of ruptured blood vessels that does not cross the suture lines 4. Accumulation of fluid resulting from partial blockage of cerebrospinal fluid drainage

2. Swelling of the soft tissue of the scalp as a result of pressure during labor

A client has an endotracheal tube and is receiving mechanical ventilation. Periodic suctioning is necessary, and the nurse follows a specific protocol when performing this procedure. Select in order of priority the nursing actions that should be taken when suctioning. 1.Insert the catheter without applying suction 2.Assess client's vital signs and lung sounds 3.Rotate the catheter while suction is applied 4.Administer oxygen via a ventilation bag

2.Assess client's vital signs and lung sounds 1.Insert the catheter without applying suction 3.Rotate the catheter while suction is applied 4.Administer oxygen via a ventilation bag

The family of a client with right ventricular heart failure expresses concern about the client's increasing abdominal girth. What physiologic change should the nurse consider when explaining the client's condition? 1. Loss of cellular constituents in blood 2. Rapid osmosis from tissue spaces to cells 3. Increased pressure within the circulatory system 4. Rapid diffusion of solutes and solvents into plasma

3. Increased pressure within the circulatory system

The laboratory report of a client reveals increased levels of atrial natriuretic peptide. Which other finding does the nurse anticipate to find in the client? 1. Decreased urine output 2. Increased concentration of urine 3. Increased sodium excretion in urine 4. Decreased glomerular filtration rate

3. Increased sodium excretion in urine

The nurse is caring for a client who is on a low-carbohydrate diet. With this diet, there is decreased glucose available for energy and fat is metabolized for energy, resulting in an increased production of which substance in the urine? 1. Protein 2. Glucose 3. Ketones 4. Uric acid

3. Ketones

A client is admitted to the postanesthesia care unit after a segmental resection of the right lower lobe of the lung. A chest tube drainage system is in place. When caring for this tube, what should the nurse do? 1. Raise the drainage system to bed level and check its patency 2. Clamp the tube when moving the client from the bed to a chair 3. Mark the time and fluid level on the side of the drainage chamber 4. Secure the chest catheter to the wound dressing with a sterile safety pin

3. Mark the time and fluid level on the side of the drainage chamber

The parents of a toddler with newly diagnosed cystic fibrosis ask a nurse what causes the problems related to this disorder. What should the nurse consider about the primary pathologic process before responding? 1. Hyperactivity of the eccrine (sweat) glands 2. Hypoactivity of the autonomic nervous system 3. Mechanical obstruction of mucus-secreting glands 4. Atrophic changes in the mucosal lining of the intestines

3. Mechanical obstruction of mucus-secreting glands

The nurse is caring for a client who has a lesion in the right upper lobe. A diagnosis of tuberculosis (TB) has been made. What are the clinical manifestations of tuberculosis? 1. Frothy sputum and fever 2. Dry cough and pulmonary congestion 3. Night sweats and blood-tinged sputum 4. Productive cough and engorged neck veins

3. Night sweats and blood-tinged sputum

Which order of actions should a nurse follow when performing a chest examination of a client with a pulmonary disorder? 1. Monitor the respiratory rate. 2. Observe for any abnormalities of the sternum. 3. Observe for any evidence of respiratory distress. 4. Observe the shape and symmetry of the chest

3. Observe for any evidence of respiratory distress.

An obese smoker complains of feeling sleepy during the daytime, waking up tired in the morning, and snoring heavily while sleeping. The client is found to have enlarged tonsils. Which condition may the client have? 1. Laryngeal trauma 2. Vocal cord paralysis 3. Obstructive sleep apnea 4. Subcutaneous emphysema

3. Obstructive sleep apnea

A nurse is admitting a 2-year-old toddler who ingested half of a bottle of aspirin tablets to the emergency department. What is the origin of the metabolic acidosis caused by aspirin toxicity? 1. Deep rapid breathing 2. Higher pH of gastric contents 3. Rapid absorption of salicylate 4. Increased renal excretion of bicarbonate

3. Rapid absorption of salicylate

A client's severe anxiety and panic are often considered "contagious." What action should be taken when a nurse's personal feelings of anxiety are increasing? 1. Refocusing the conversation to more pleasant topics 2. Saying to the client, "Calm down. You're making me anxious, too." 3. Saying, "Another staff member is coming in. I'll leave and come back later." 4. Remaining quiet so personal feelings of anxiety do not become apparent to the client

3. Saying, "Another staff member is coming in. I'll leave and come back later."

The parents of a school-aged child with leukemia ask the nurse why irradiation of the spine and skull is necessary. What is the most accurate response by the nurse? 1. "Radiation retards the growth of cells in the bone marrow of the cranium." 2. "This therapy decreases cerebral edema and prevents increased intracranial pressure." 3. "Leukemic cells may invade the nervous system, but the usual drugs are ineffective in the brain." 4. "Neoplastic drug therapy without radiation is effective in most cases, but this is a precautionary treatment."

3. "Leukemic cells may invade the nervous system, but the usual drugs are ineffective in the brain."

During assessment, the nurse asks a client about developmental milestones such as the age at which thelarche and menarche occurred. The nurse determines that the client experienced pubertal delay. Which finding in the client's history supports the nurse's conclusion? 1. Weight increased by 8 to 12 kg. 2. Menarche occurred 2 years after thelarche. 3. Breast development occurred by 15 years of age. 4. Growth in height stopped 2 years after menarche.

3. Breast development occurred by 15 years of age.

A 62-year-old client reports to the nurse, "My eyes don't feel right and I have a gritty and sandy sensation in my eyes." What condition might this client have? 1. Retinal detachment 2. Infection of the cornea 3. Changes in tear composition 4. Hemorrhage in the vitreous humor

3. Changes in tear composition

A client is admitted to the hospital with atrial fibrillation. A diagnosis of mitral valve stenosis is suspected. The nurse concludes that it is most significant if the client presents with what history? 1. Cystitis as an adult 2. Pleurisy as an adult 3. Childhood strep throat 4. Childhood German measles

3. Childhood strep throat

The nurse is reviewing the laboratory reports of a client who has sustained a significant reaction to the tuberculin skin test but has negative findings on bacteriologic studies. The reports further reveal the absence of x-ray findings compatible with tuberculosis (TB) and clinical evidence of TB. Which class of TB does the nurse suspect? 1. Class 0 2. Class 1 3. Class 2 4. Class3

3. Class 2

A hospice nurse is caring for a dying client and the client's family members during the developing awareness stage of grief. What is the most important thing about the family that the nurse should assess before providing care? 1. Cohesiveness 2. Educational level 3. Cultural background 4. Socioeconomic status

3. Cultural background

In her eighth month of pregnancy, a 24-year-old client is brought to the hospital by the police, who were called when she barricaded herself in a ladies' restroom of a restaurant. During admission the client shouts, "Don't come near me! My stomach is filled with bombs, and I'll blow up this place if anyone comes near me." What does the nurse conclude that the client is exhibiting? 1. Ideas of reference 2. Loose associations 3. Delusional thinking 4. Tactile hallucinations

3. Delusional thinking

A nurse is volunteering on the community crisis hotline. What is the final objective of the counseling process? 1. Reducing anxiety 2. Exploring feelings 3. Developing constructive coping skills 4. Accomplishing the debriefing process

3. Developing constructive coping skills

A pediatric client reports difficulty breathing and swallowing and has a sore throat, headache, and fever. The nurse observes a grayish yellow membranous patch near the tonsils. What disease does the nurse infer from these findings? 1. Mumps 2. Pertussis 3. Diphtheria 4. Pneumonia

3. Diphtheria

A nurse is caring for a client exhibiting compulsive behaviors. The nurse concludes that the compulsive behavior usually incorporates the use of which defense mechanism? 1. Projection 2. Regression 3. Displacement 4. Rationalization

3. Displacement

What role is the nurse expected to have in a community-based nursing practice if there is a sudden spread of malaria? 1. Educator 2. Collaborator 3. Epidemiologist 4. Client advocate

3. Epidemiologist

After assessing a 5-year-old child, the nurse suspects the child has mumps. Which symptoms present in the child may support the nurse's suspicion? Select all that apply. 1. Skin rash 2. Sore throat 3. Fever (100° F-104° F [37.8°-40° C]) 4. Difficulty swallowing 5. Swelling of the parotid gland

3. Fever (100° F-104° F [37.8°-40° C]) 5. Swelling of the parotid gland

A client is admitted to the hospital with multiple signs and symptoms associated with a cardiac problem. What clinical finding alerts the nurse that the primary healthcare provider probably will insert a pacemaker? 1. Angina 2. Chest pain 3. Heart block 4. Tachycardia

3. Heart block

A client is admitted to a psychiatric hospital with the diagnosis of schizoid personality disorder. Which initial nursing intervention is a priority for this client? 1. Helping the client enter into group recreational activities 2. Convincing the client that the hospital staff is trying to help 3. Helping the client learn to trust the staff through selected experiences 4. Limiting the client's contact with others while in the hospital

3. Helping the client learn to trust the staff through selected experiences

A client is found to have a borderline personality disorder. What behavior does the nurse consider is most typical of these clients? 1.Inept 2. Eccentric 3. Impulsive 4. Dependent

3. Impulsive

At 1 am a 28-month-old toddler is admitted to the pediatric unit with suspected meningitis. At 3 am, after the child is settled in, the mother tells the nurse, "I have to leave now, but whenever I try to go my child gets upset and then I start to cry." What is the best action by the nurse? 1. Walking the mother to the elevator 2. Encouraging the mother to spend the night 3. Staying with the child while the mother leaves 4. Telling the mother to wait until the child falls asleep

3. Staying with the child while the mother leaves

A nurse in the emergency department is assessing a client who has been physically and sexually assaulted. What is the nurse's priority during assessment? 1. The family's feelings about the attack 2. The client's feelings of social isolation 3. The client's ability to cope with the situation 4. Disturbance in the client's thought processes

3. The client's ability to cope with the situation

When visiting hours are over, a nurse approaches a client with paranoid schizophrenia, who shouts, "You're the one that made my lover leave me." What conclusion does the nurse make about the client? 1. The patient is disoriented. 2. The patient is actively hallucinating. 3. The patient feels a sense of vulnerability. 4. The patient needs to have limits set after calming down.

3. The patient feels a sense of vulnerability.

A client is to receive conscious sedation during a cardiac catheterization. Which route of administration should the nurse explain will be used to deliver the conscious sedation? 1. Via a face mask 2. Into the epidural space 3. Through an intravenous catheter 4. Around the nerves innervating the chest wall

3. Through an intravenous catheter

During a group discussion it is learned that a group member hid suicidal urges and committed suicide several days ago. What should the nurse leading the group be prepared to manage? 1. Guilt of the co-leaders for failing to anticipate and prevent the suicide 2. Guilt of group members because they could not prevent another's suicide 3. Lack of concern over the suicide expressed by several of the members in the group 4. Fear by some members that their own suicidal urges may go unnoticed and that they may go unprotected

4. Fear by some members that their own suicidal urges may go unnoticed and that they may go unprotected

During the course of treatment a toddler is to receive an intramuscular injection. What is the priority nursing intervention that should be included in the plan of care to comfort the child? 1. Distracting the toddler's attention with a toy car 2. Telling the parents exactly what will be done to the toddler 3. Giving the toddler the choice of having the injection now or later 4. Involving the parents in comforting the toddler after the injection

4. Involving the parents in comforting the toddler after the injection

Which statement is true regarding varicocele? 1. It is commonly seen in prepubertal children. 2. It results in partial or complete venous occlusion. 3. It results in the red, warm, and edematous scrotum. 4. It causes elongation of the veins of the spermatic cord.

4. It causes elongation of the veins of the spermatic cord.

The nurse is caring for a client who is admitted with a crushing injury to the spinal cord above the level of phrenic nerve origin. What should the nurse consider about this type of injury when planning care? 1. Ventricular fibrillation 2. Vagus nerve dysfunction 3. Retention of sensation and paralysis of lower extremities 4. Lack of diaphragmatic contractions and respiratory paralysis

4. Lack of diaphragmatic contractions and respiratory paralysis

Endotracheal intubation and positive-pressure ventilation are instituted because of a client's deteriorating respiratory status. Which is the priority nursing intervention? 1. Facilitate verbal communication 2. Prepare the client for emergency surgery 3. Maintain sterility of the ventilation system 4. Assess the client's response to the mechanical ventilation

4. Assess the client's response to the mechanical ventilation

The nurse is assessing a term newborn. Which sign should the nurse report to the pediatric primary healthcare provider? 1. Temperature of 97.7° F (36.5° C) 2. Pale pink to rust-colored stain in the diaper 3. Heart rate that decreases to 115 beats/min 4. Breathing pattern with recurrent sternal retractions

4. Breathing pattern with recurrent sternal retractions

A client is admitted with head trauma after a fall. The client is being prepared for a supratentorial craniotomy with burr holes, and an intravenous infusion of mannitol is instituted. The nurse concludes that this medication primarily is given to do what? 1. Lower blood pressure 2. Prevent hypoglycemia 3. Increase cardiac output 4. Decrease fluid in the brain

4. Decrease fluid in the brain

The nurse is caring for a 70-year-old client who presents with dilute urine even when fluid intake is low. What could be the possible cause of the client's condition? 1. Decreased glucose tolerance 2. Decreased general metabolism 3. Decreased ovarian production of estrogen 4. Decreased antidiuretic hormone production

4. Decreased antidiuretic hormone production

Which suggestion should the nurse offer to parents who are concerned about caring for their toddler? 1. Refrain from giving more than five cups of milk a day. 2. Allow the toddler to choose a time to take medicine. 3. Let the toddler watch television if the parent is busy. 4. Encourage the toddler to drink from two-handled cups.

4. Encourage the toddler to drink from two-handled cups.

A 16-year-old boy with a diagnosis of adolescent adjustment disorder and his family are beginning family therapy. What is the best initial nursing approach? 1. Setting long-term goals for the family 2. Letting the client express his feelings first 3. Having the parents explain their rationale for setting firm limits 4. Encouraging each family member to share how the problem is perceived

4. Encouraging each family member to share how the problem is perceived

While assessing the client's skin, a nurse notices a skin condition, the pathophysiology of which involves increased visibility of oxyhemoglobin caused by an increased blood flow due to capillary dilation. Which condition is associated with this client? 1. Pallor 2. Vitiligo 3. Cyanosis 4. Erythema

4. Erythema

What is the priority nursing objective of the therapeutic psychiatric environment for a confused client? 1. Helping the client relate to others 2. Making the hospital atmosphere more homelike 3. Helping the client become accepted in a controlled setting 4. Maintaining the highest level of safe, independent function

4. Maintaining the highest level of safe, independent function

An adolescent who works out 6 hours a day reports not eating well, weight loss, and an absence of menses for the past few months. Which nursing intervention is most appropriate? 1. Ask the adolescent to stop exercising for a few days. 2. Talk to the client to find out any reasons for stress. 3. Perform a β-human chorionic gonadotropin pregnancy test. 4. Modify the adolescent's diet to incorporate more nutrients.

4. Modify the adolescent's diet to incorporate more nutrients.

A 1-month-old infant with a ventricular septal defect (VSD) is examined in the cardiology clinic. What sign related to this disorder does the nurse expect to find when assessing this infant? 1. Bradycardia at rest 2. Activity-related cyanosis 3. Bounding peripheral pulses 4. Murmur at the left sternal border

4. Murmur at the left sternal border

The nurse is caring for a client who had a colostomy 2 days ago. Which nursing intervention is the priority? 1. Keeping an accurate record of oral fluid intake 2. Emphasizing the importance of regulating the diet to form stool 3. Teaching care of the incision and how to perform colostomy irrigations 4. Observing for drainage and the condition of the abdominal stoma

4. Observing for drainage and the condition of the abdominal stoma

An expectant couple asks the nurse about the cause of low back pain in labor. The nurse replies that this pain occurs most often when the fetus is in what position? 1. Breech 2. Transverse 3. Occiput anterior 4. Occiput posterior

4. Occiput posterior

The nurse is assessing the rate of involution of a client's uterus on the second postpartum day. Where does the nurse expect the fundus to be located? 1. At the level of the umbilicus 2. One fingerbreadth above the umbilicus 3. Above and to the right of the umbilicus 4. One or two fingerbreadths below the umbilicus

4. One or two fingerbreadths below the umbilicus

A school-aged child who has just arrived from Africa has been exposed to diphtheria, and a nurse in the pediatric clinic is to administer the antitoxin. Which type of immunity does the antitoxin confer? 1. Active natural 2. Passive natural 3. Active artificial 4. Passive artificial

4. Passive artificial

When providing preoperative teaching, what should the nurse focus primarily on? 1. Helping the client and family decide if surgery is necessary 2. Providing emotional support to the client and family 3. Giving minute-by-minute details of the surgery to the client and family 4. Providing general information to reduce client and family anxiety

4. Providing general information to reduce client and family anxiety

A client who sustained burn injuries due to a fire and explosion has a carbon monoxide level of 14%. Which pathophysiologic risk is increased in the client? 1. Stupor 2.. Vertigo 3. Convulsions 4. Slight breathlessness

4. Slight breathlessness

A nurse is caring for a client who is cachectic. What information about the function of adipose tissue in fat metabolism is necessary to better address the needs of this client? 1. Releases glucose for energy 2. Regulates cholesterol production 3. Uses lipoproteins for fat transport 4. Stores triglycerides for energy reserves

4. Stores triglycerides for energy reserves

When a client has a myocardial infarction, one of the major manifestations is a decrease in the conductive energy provided to the heart. When assessing this client, the nurse is aware that the existing action potential is in direct relationship to what? 1. Heart rate 2. Refractory period 3. Pulmonary pressure 4. Strength of contraction

4. Strength of contraction

A 60-year-old woman is admitted for a vaginal hysterectomy and anterior and posterior repair of the vaginal wall. While taking the nursing history the nurse expects the client to state that one of the reasons she is having surgery is because she has been experiencing what? 1. Hematuria 2. Dysmenorrhea 3. Pain on urination 4. Stress incontinence

4. Stress incontinence

What should the nurse include in the plan of care for a client who just had a total laryngectomy? 1. Instructing the client to whisper 2. Removing the outer tracheostomy tube as needed 3. Placing the client in the orthopneic position 4. Suctioning the tracheostomy tube whenever necessary

4. Suctioning the tracheostomy tube whenever necessary


Kaugnay na mga set ng pag-aaral

Finance Chapter 7 Practice Problems

View Set

Sadlier-Oxford Vocab Level H - Unit 11

View Set

IM - CHF & EKG, pericarditis, cardiac tamponade, atheroembolism, MVP, Aortic regurg

View Set

Unit 4 - Types of Life Insurance Policies

View Set

Chemistry A Volatile History Test Episode 2

View Set